K
Khách

Hãy nhập câu hỏi của bạn vào đây, nếu là tài khoản VIP, bạn sẽ được ưu tiên trả lời.

AH
Akai Haruma
Giáo viên
7 tháng 2 2020

Lời giải:

$A=a-\frac{ac}{c+a^2}+b-\frac{ab}{a+b^2}+c-\frac{bc}{b+c^2}$

$=\sum a-\sum \frac{ac}{c+a^2}$

Áp dụng BĐT AM-GM: $c+a^2\geq 2a\sqrt{c}$

$\Rightarrow A\geq \sum a-\frac{1}{2}\sum \sqrt{c}$

Áp dụng BĐT Cauchy-Schwarz:

$(\sum \sqrt{c})^2\leq (c+a+b)(1+1+1)$

$\Rightarrow \sum \sqrt{c}\leq 3\sum a$

Do đó $A\geq \sum a-\frac{1}{2}\sqrt{3\sum a}$

Đặt $\sqrt{3\sum a}=t$ thì $A\geq \frac{t^2}{3}-\frac{t}{2}(*)$

Từ điều kiện $ab+bc+ac=3abc\Rightarrow 3=\frac{1}{a}+\frac{1}{b}+\frac{1}{c}$

Áp dụng BĐT Cauchy-Schwarz:

$3=\sum \frac{1}{a}\geq \frac{9}{\sum a}\Rightarrow \sum a\geq 3$

$\Rightarrow t=\sqrt{3\sum a}\geq 3$

Do đó:

$\frac{t^2}{3}-\frac{t}{2}=(t-3)(\frac{t}{3}+\frac{1}{2})+\frac{3}{2}\geq \frac{3}{2}$ với mọi $t\geq 3(**)$

Từ $(*); (**)\Rightarrow A\geq \frac{3}{2}$

Vậy $A_{\min}=\frac{3}{2}$ khi $a=b=c=1$

8 tháng 1 2021

Hi vọng là tìm GTLN:

Không mất tính tổng quát, giả sử b, c cùng phía với 1 \(\Rightarrow\left(b-1\right)\left(c-1\right)\ge0\Leftrightarrow bc\ge b+c-1\).

Áp dụng bất đẳng thức AM - GM ta có: 

\(4=a^2+b^2+c^2+abc\ge a^2+2bc+abc\Leftrightarrow2bc+abc\le4-a^2\Leftrightarrow bc\left(a+2\right)\le\left(2-a\right)\left(a+2\right)\Leftrightarrow bc+a\le2\)

\(\Rightarrow a+b+c\le3\).

Áp dụng bất đẳng thức Schwarz ta có:

\(P\le\dfrac{ab}{9}\left(\dfrac{1}{a}+\dfrac{2}{b}\right)+\dfrac{bc}{9}\left(\dfrac{1}{b}+\dfrac{2}{c}\right)+\dfrac{ca}{9}\left(\dfrac{1}{c}+\dfrac{2}{a}\right)=\dfrac{1}{9}.3\left(a+b+c\right)=\dfrac{1}{3}\left(a+b+c\right)\le1\).

Đẳng thức xảy ra khi a = b = c = 1.

8 tháng 1 2021

đề là tìm GTNN ạ, dù gì cũng cảm ơn bạn nha <3

AH
Akai Haruma
Giáo viên
11 tháng 5 2019

Lời giải:
Vì $abc=1$ nên:

\((a+bc)(b+ac)(c+ab)=a(a+bc)b(b+ac)c(c+ab)=(a^2+1)(b^2+1)(c^2+1)\)

Áp dụng BĐT Bunhiacopxky:

\((a^2+1)(1+b^2)\geq (a+b)^2; (a^2+1)(1+c^2)\geq (a+c)^2; (b^2+1)(1+c^2)\geq (b+c)^2\)

Nhân theo vế và thu gọn:

\(\Rightarrow (a^2+1)(b^2+1)(c^2+1)\geq (a+b)(b+c)(c+a)\)

Lại có: Theo BĐT AM-GM thì:

\((a+b)(b+c)(c+a)=(ab+bc+ac)(a+b+c)-abc\)

\(\geq (ab+bc+ac)(a+b+c)-\frac{(a+b+c)(ab+bc+ac)}{9}=\frac{8(a+b+c)(ab+bc+ac)}{9}(*)\) (đây là BĐT khá quen thuộc rồi)

Do đó:

\(P=\frac{(a+bc)(b+ca)(c+ab)}{ab+bc+ac}+\frac{1}{a+b+c}=\frac{(a^2+1)(b^2+1)(c^2+1)}{ab+bc+ac}+\frac{1}{a+b+c}\geq \frac{(a+b)(b+c)(c+a)}{ab+bc+ac}+\frac{1}{a+b+c}\)

\(P\geq \frac{7(a+b)(b+c)(c+a)}{8(ab+bc+ac)}+\frac{(a+b)(b+c)(c+a)}{8(ab+bc+ac)}+\frac{1}{a+b+c}\)

Áp dụng BĐT (*) và AM-GM:

\(\frac{7(a+b)(b+c)(c+a)}{8(ab+bc+ac)}\geq 7.\frac{\frac{8}{9}(a+b+c)(ab+bc+ac)}{8(ab+bc+ac)}=\frac{7}{9}(a+b+c)\geq \frac{7}{9}.3\sqrt[3]{abc}=\frac{7}{3}\)

\(\frac{(a+b)(b+c)(c+a)}{8(ab+bc+ac)}+\frac{1}{a+b+c}\geq 2\sqrt{\frac{(a+b)(b+c)(c+a)}{8(ab+bc+ac)(a+b+c)}}\geq 2\sqrt{\frac{\frac{8}{9}(a+b+c)(ab+bc+ac)}{8(a+b+c)(ab+bc+ac)}}=\frac{2}{3}\)

\(\Rightarrow P\geq \frac{7}{3}+\frac{2}{3}=3\)

Vậy $P_{\min}=3$

12 tháng 5 2019

\(\left(a+bc\right)\left(b+ca\right)\left(c+ab\right)\)

\(=a^2+b^2+c^2+a^2b^2+b^2c^2+c^2a^2+1+1\)

\(=a^2+b^2+c^2+a^2b^2+b^2c^2+c^2a^2+1+1+1-1\)

Áp dụng BĐT AM-GM ta có:

\(\left(a+bc\right)\left(b+ca\right)\left(c+ab\right)\ge a^2+b^2+c^2+2ab+2bc+2ac-1=\left(a+b+c\right)^2-1\)\(\Rightarrow P\ge\frac{\left(a+b+c\right)^2-1}{ab+bc+ca}+\frac{1}{a+b+c}\)

Dấu " = " xảy ra <=> ...

Ta có: \(\frac{1}{3}.\left(a+b+c\right)^2\ge ab+bc+ca\)( BĐT quen thuộc tự c/m)

\(\Rightarrow P\ge\frac{\left(a+b+c\right)^2-1}{ab+bc+ca}+\frac{1}{a+b+c}\ge\frac{\left(a+b+c\right)^2}{\frac{1}{3}\left(a+b+c\right)^2}-\frac{1}{\frac{1}{3}\left(a+b+c\right)}+\frac{1}{a+b+c}\)\(=3+\frac{a+b+c-3}{\left(a+b+c\right)^2}\)

Ta có: \(abc=1\Leftrightarrow\sqrt[3]{abc}=1\le\frac{a+b+c}{3}\left(AM-GM\right)\)

\(\Rightarrow a+b+c\ge3\)

Dấu " = " xảy ra <=> ...

\(\Rightarrow P\ge3+\frac{a+b+c-3}{\left(a+b+c\right)^2}\ge3\)

Dấu " = " xảy ra <=> a=b=c=1

KL:...........

NV
13 tháng 6 2021

\(\sqrt{\dfrac{ab}{c+ab}}=\sqrt{\dfrac{ab}{c\left(a+b+c\right)+ab}}=\sqrt{\dfrac{ab}{\left(a+c\right)\left(b+c\right)}}\le\dfrac{1}{2}\left(\dfrac{a}{a+c}+\dfrac{b}{b+c}\right)\)

Tương tự: \(\sqrt{\dfrac{bc}{a+bc}}\le\dfrac{1}{2}\left(\dfrac{b}{a+b}+\dfrac{c}{a+c}\right)\) ; \(\sqrt{\dfrac{ca}{b+ca}}\le\dfrac{1}{2}\left(\dfrac{a}{a+b}+\dfrac{c}{b+c}\right)\)

Cộng vế với vế:

\(P\le\dfrac{1}{2}\left(\dfrac{a}{a+c}+\dfrac{c}{a+c}+\dfrac{b}{b+c}+\dfrac{c}{b+c}+\dfrac{b}{a+b}+\dfrac{a}{a+b}\right)=\dfrac{3}{2}\)

Dấu "=" xảy ra khi \(a=b=c=\dfrac{1}{3}\)

1 tháng 9 2021

Cho a, b, c, d là các chữ số thỏa mãn: ab+ca=da ab-ca=a Tìm giá trị của d.